flux!
- 32
- 0
Homework Statement
How do I find the total charge from a material with a charge density given by
\rho =10^{-9} \text{cos}\left ( \frac{z}{z_0}\right ) C/m^3
that exist between \frac{-\pi}{3}z_0<z<\frac{\pi}{3}z_0.
Homework Equations
None I can think of.
The Attempt at a Solution
Attempt #1:
Since the charge density is a volume charge density we may assume that we are dealing with a cylindrical charge distribution that remains the same along r. However, the radius of the assumed shaped was not given, so let us assume that its radius is r_0. We may solve this by:
q=\int_V \rho d\tau q=\int_0^{r_0} \int_0^{2\pi} \int_{\frac{-\pi}{3}z_0}^{\frac{\pi}{3}z_0} 10^{-9} \text{cos}\left ( \frac{z}{z_0}\right ) C/m^3 rdrd\phi dz
Everything will then be straight forward, but the issue is that
1. r_0 is not given, and
2. the problem did not say it is a cylindrical charge density.
This is just, however, one way I could deal with a volume charge density that does not have a given volume, unusual huh.
Attempt #2:
Since the problem indicated that the volume charge density is distributed on a line, then it must be a line charge only. Thus it must have an infinitesimal radius that could be resolve by Dirac Delta function, so:
\rho =10^{-9} \text{cos}\left ( \frac{z}{z_0}\right ) C/m^3
must be equivalent to:
\rho =\delta(r,\phi - r',\phi')10^{-9} \text{cos}\left ( \frac{z}{z_0}\right ) C/m
then, things could now be easily solved in the integral:
q=\int_V \left (\delta(r,\phi - r',\phi')10^{-9} \text{cos}\left ( \frac{z}{z_0}\right ) C/m\right )d\tau
4. My Question
Now, my question is, which two approaches is the right solution, Or if neither any of the two is correct, how should we solve the total charge of a density given above?
Cross-Link: Posted the Question In physics.exchange but, does not seem to get answered, Hope you guys help me out. http://physics.stackexchange.com/questions/240215/how-to-compute-the-charge-of-a-density-distributed-along-z-axis
Last edited by a moderator: